1 Introduction

A smooth Fano threefold of Picard rank 2 and degree 28 is the blow-up of a smooth quadric threefold \(Q\subset \mathbb {P}^4\) in a smooth rational quartic curve \(C_4\subset Q\). Isomorphism classes of such threefolds form an irreducible two-dimensional family, which according to the Mori-Mukai classification corresponds to family 2.21. Let \(\pi :X\rightarrow Q\) be such a threefold. Then the action of \(\text {Aut}(Q,C_4)\) on Q lifts to an action on X, so that we may identify it with a subgroup of \(\text {Aut}(X)\).

By a result of Cheltsov-Przyjalkowski-Shramov ([3, Lemma 9.2]), we have that either \(\text {Aut}(X)\) is finite, or \(\text {Aut}(X)\cong \text {Aut}(Q,C_4)\times \mathbb {Z}_2\), where upto isomorphism \(\text {Aut}(Q,C_4)\) is described as follows:

  1. (1)

    There is a unique smooth threefold in family 2.21, unique upto isomorphism, such that \(\text {Aut}(Q,C_4)\cong \textrm{PGL}_2(\mathbb {C})\),

  2. (2)

    There is a one-dimensional family of non-isomorphic smooth threefolds in family 2.21 such that \(\text {Aut}(Q,C_4)\cong \mathbb {G}_m\rtimes \mathbb {Z}_2\),

  3. (3)

    There is a unique smooth threefold in family 2.21 such that \(\text {Aut}(Q,C_4)\cong \mathbb {G}_a\rtimes \mathbb {Z}_2\).

The goal of this paper is to describe \(\text {Aut}(X)\) when it is finite, where X is a smooth threefold in family 2.21. Our main result is the following:

Theorem 1.1

Let X be a smooth Fano threefold of rank 2 and degree 28. Then \(\textrm{Aut}(X)\cong \textrm{Aut}(Q,C_4)\times \mathbb {Z}_2\). Furthermore, if \(\textrm{Aut}(Q,C_4)\) is finite then it is isomorphic to \(\mathbb {Z}_2\times \mathbb {Z}_2,\mathbb {Z}_2\) or 0.

We prove this theorem in two parts: Theorem 2.1 and Theorem 3.1.

Remark 1.2

The factor of \(\mathbb {Z}_2\) appearing in the factorisation \(\textrm{Aut}(X)\cong \textrm{Aut}(Q,C_4)\times \mathbb {Z}_2\) is generated by an involution g, which may be described as follows:

Let \(\mathfrak {d}\) denote the restriction to Q of the linear system of quadric hypersurfaces in \(\mathbb {P}^4\) which contain \(C_4\), and let \(\phi :Q\dashrightarrow \mathbb {P}^4\) be the corresponding rational map. The image of \(\phi \) is a smooth quadric threefold, and \(\phi \) contracts the intersection of the secant variety of \(C_4\) with Q, V, onto a smooth rational curve \(C_4'\subset Q'\). The base locus of \(\phi \) is equal to \(C_4\), and there is a birational morphism \(\pi ':X\rightarrow Q'\), where X is the blow-up of Q along \(C_4\). This morphism contracts the strict transform, \(E'\), of V onto the curve \(C_4'\). Thus, there is a commutative diagram

figure a

In [3], it is shown that in cases (1) and (2) of the above classification, there exists a basis of \(\mathfrak {d}\) such that \(Q'=Q\) and \(C_4'=C_4\), so that \(\phi \) lifts to an involution \(g\in \textrm{Aut}(X)\). We will show in Theorem 3.1 that this is always the case.

We can explicitly describe the threefolds appearing in [3, Lemma 9.2]. Let us fix some notation. Observe that after a projective transformation \(C_4\) is the image of the Veronese embedding of \(\mathbb {P}^1\) in \(\mathbb {P}^4\):

$$\begin{aligned} \mathbb {P}^1&\rightarrow \mathbb {P}^4 \\ [u:v]&\mapsto [u^4:u^3v:u^2v^2:uv^3:v^4]. \end{aligned}$$

The space of global sections of \(\mathcal {I}_{C_4}(2)\) is generated by the following quadratic forms:

$$\begin{aligned} f_0&= x_3^2 - x_2x_4, \\ f_1&= x_2x_3 - x_1x_4,\\ f_2&= x_2^2 - x_0x_4,\\ f_3&= x_1x_2 - x_0x_3,\\ f_4&= x_1^2 - x_0x_2,\\ f_5&= 3x_2^2 - 4x_1x_3 + x_0x_4, \end{aligned}$$

where \(\mathcal {I}_{C_4}\) is the ideal sheaf of \(C_4\) in \(\mathbb {P}^4\), and \(x_0\), \(x_1\), \(x_2\), \(x_3\), \(x_4\) are homogeneous coordinates on \(\mathbb {P}^4\). Observe that the standard \(\textrm{PGL}_2(\mathbb {C})\)-action on \(C_4\) lifts to an action on \(\mathbb {P}^4\) such that \(C_4\) is invariant. We fix the following subgroups of \(\textrm{PGL}_2(\mathbb {C})\):

$$\begin{aligned}&\mathbb {Z}_2,\text { generated by } \begin{pmatrix}0 &{} 1 \\ 1 &{} 0\end{pmatrix},\\&\mathbb {G}_m,\text { consisting of matrices } \begin{pmatrix}1 &{} 0 \\ 0 &{} t\end{pmatrix} \text { for every } t\in \mathbb {G}_m,\\&\mathbb {G}_a,\text { consisting of matrices } \begin{pmatrix}1 &{} t \\ 0 &{} 1\end{pmatrix} \text { for every } t\in \mathbb {G}_a. \end{aligned}$$

Now we can describe \(\textrm{Aut}(X)\) for the threefolds listed before:

Example 1.3

([1, Section 5.9]). Let Q be the quadric given by the equation

$$\begin{aligned}(1-4s^2)f_2+f_5=0,\end{aligned}$$

for some \(s\in \mathbb {C}\setminus \{-1,0,1\}\). Then Q is \(\mathbb {G}_m\)-invariant and \(\mathbb {Z}_2\)-invariant, and conversely any smooth quadric admitting a faithful \(\mathbb {G}_m\)-action is isomorphic, via an element of \(\textrm{PGL}_2(\mathbb {C})\), to a quadric given by an equation of this form. Moreover, we have the following:

$$\begin{aligned}\textrm{Aut}(Q,C_4)\cong {\left\{ \begin{array}{ll} \mathbb {G}_m\rtimes \mathbb {Z}_2, &{} s\ne \pm \frac{1}{2}, \\ \textrm{PGL}_2(\mathbb {C}), &{} s=\pm \frac{1}{2}. \end{array}\right. }\end{aligned}$$

The involution g described before is given by:

$$\begin{aligned}\tau :[x_0:x_1:x_2:x_3:x_4]\mapsto [f_4:sf_3:s^2f_2:sf_1:f_0].\end{aligned}$$

See [1, Remark 5.52] for an explanation of why \(\tau \circ \tau :Q\dashrightarrow Q\) is the identity map on \(Q\setminus C_4\).

Example 1.4

Suppose that the quadric Q is given by the equation

$$\begin{aligned}f_0+f_5=0.\end{aligned}$$

Then Q is \(\mathbb {G}_a\)-invariant and \(\mathbb {Z}_2\)-invariant, and \(\textrm{Aut}(Q,C_4)\cong \mathbb {G}_a\rtimes \mathbb {Z}_2\). We will prove in case (2) of Theorem 3.1 that the blow-up of Q in \(C_4\) admits an action of the involution g.

Remark 1.5

Recall that for a finite subgroup \(G\subset \text {Aut}(Y)\), a variety Y is called G-Fano if it has terminal singularities, \(-K_Y\) is ample and \(\text {Cl}(Y)^G\) is rank 1. It is proven in [4] that the Hilbert scheme of conics on a smooth threefold X from the family No2.21 is isomorphic to \(\mathbb {P}^1\times \mathbb {P}^1\), with the degenerate conics being parameterised by a smooth curve \(C\subset \mathbb {P}^1\times \mathbb {P}^1\) of bidegree (2, 2). If X is G-Fano for the group \(G=\langle g\rangle \cong \mathbb {Z}_2\), then this curve must be invariant upon swapping the two factors of \(\mathbb {P}^1\).

An informal conjecture of Y. Prokhorov is that the invariance of this curve is a sufficient condition for X to be G-Fano. It is proven in [2] that every smooth curve in \(\mathbb {P}^1\times \mathbb {P}^1\) of bidegree (2, 2) is invariant. As a corollary to Theorem 1.1, we have that every smooth threefold X in the family No2.21 is G-Fano, so that Prokhorov’s informal conjecture is true. For a detailed discussion of G-Fano threefolds, see [6].

Remark 1.6

Smooth threefolds in the family No2.21 are parametrised by \(\mathbb {P}^5\setminus \Delta \), where \(\Delta \subset \mathbb {P}^5\) is the discriminant locus of singular quadrics. The group \(\textrm{PGL}_2(\mathbb {C})\) acts on this space, and it follows from Theorem 1.1 that any two threefolds in family 2.21 are isomorphic if and only if their corresponding points in the parameter space \(\mathbb {P}^5\setminus \Delta \) lie in the same \(\textrm{PGL}_2(\mathbb {C})\)-orbit. Moreover, the moduli space of smooth GIT-polystable threefolds in family 2.21 is given by the GIT quotient

$$\begin{aligned}(\mathbb {P}^5\setminus \Delta )//\textrm{PGL}_2(\mathbb {C}).\end{aligned}$$

2 Computation of \(\text {Aut}(Q,C_4)\)

The first half of proving Theorem 1.1 is the computation of \(\textrm{Aut}(Q,C_4)\), which we will do in this section. The result we will prove is:

Theorem 2.1

Let Q be a smooth quadric threefold containing the quartic curve \(C_4\). If \(\textrm{Aut}(Q,C_4)\) is finite, then it is isomorphic to either \(\mathbb {Z}_2\times \mathbb {Z}_2\), \(\mathbb {Z}_2\) or 0.

The following lemma will be useful:

Lemma 2.2

Let \(Q\subset \mathbb {P}^4\) be any quadric hypersurface containing the curve \(C_4\). Suppose that \(\textrm{Aut}(Q,C_4)\) is finite, and contains an element of finite order \(n>2\). Then Q is singular.

Proof

Since \(\textrm{Aut}(Q,C_4)\subseteq \textrm{Aut}(\mathbb {P}^4,C_4)\cong \textrm{PGL}_2(\mathbb {C})\), we may identify \(\textrm{Aut}(Q,C_4)\) with a subgroup of \(\textrm{PGL}_2(\mathbb {C})\). Moreover, by considering the action of \(\textrm{PGL}_2(\mathbb {C})\) on the parameter space \(\mathbb {P}^5\), we identify \(\textrm{Aut}(Q,C_4)\) with the stabiliser of the point of \(\mathbb {P}^5\) corresponding to Q. Fix a finite cyclic subgroup \(G\subset \text {PGL}_2(\mathbb {C})\) of order n, and let \(g_1\in G\) be a generator. Then \(g_1\) fixes precisely two distinct points of \(\mathbb {P}^1\), which upto projective transformation are [0 : 1] and [1 : 0]. Thus

$$\begin{aligned}g_1=\begin{pmatrix} 1 &{} 0 \\ 0 &{} \zeta \end{pmatrix},\end{aligned}$$

for some primitive \(n^\text {th}\) root of unity \(\zeta \). Then \(g_1\) acts on \(\mathbb {P}^5\) by:

$$\begin{aligned}\mapsto [\zeta ^6a_0:\zeta ^5a_1:\zeta ^4a_2:\zeta ^3a_3:\zeta ^2a_4:\zeta ^4a_5],\end{aligned}$$

and we can read off the points of \(\mathbb {P}^5\) whose stabiliser contains G:

  • \(n=2\): \((\mathbb {P}^5)^G = \big \{[a_0:0:a_2:0:a_4:a_5],[0:a_1:0:a_3:0:0]\big \}\),

  • \(n=3\): \((\mathbb {P}^5)^G = \big \{[a_0:0:0:a_3:0:0], [0:a_1:0:0:a_4:0]\big \}\),

  • \(n=4\): \((\mathbb {P}^5)^G = \big \{[a_0:0:0:0:a_4:0]\big \},\)

  • \(n>4\): \((\mathbb {P}^5)^G=\varnothing \),

where the numbers \(a_0,a_1,a_2,a_3,a_4,a_5\) are all arbitrary complex numbers. One checks that for \(n>2\), the corresponding threefolds which have finite \(\textrm{Aut}(Q,C_4)\) are all singular.

Now let us recall the following classification theorem for quadric threefolds which contain the curve \(C_4\):

Theorem 2.3

([5]) Let \(Q\subset \mathbb {P}^4\) be a smooth quadric containing \(C_4\). Then there exists an automorphism \(\phi \in \textrm{PGL}_2(\mathbb {C})\) such that \(\phi (Q)\) is given by one of the following equations:

  1. (1)

    \(\mu (f_0+f_4)+\lambda f_2+f_5=0\), for some \(\lambda \in \mathbb {C}\setminus \{1,-3\}\) and \(\mu \in \mathbb {C}\setminus \{2,-2\}\) such that \(\mu ^2\ne -\lambda ^2-2\lambda +3\),

  2. (2)

    \(f_0+\lambda f_2+ f_5=0\), for some \(\lambda \in \mathbb {C}\setminus \{1,-3\}\),

  3. (3)

    \(f_1+f_5=0\).

Let us find \(\text {Aut}(Q,C_4)\) in each of these cases.

Proof of Theorem 2.1

We may assume that \(\mu \ne 0\) in case (1), and \(\lambda \ne 0\) in case (2), as otherwise the threefolds are isomorphic to those which are described in Example 1.3 and Example 1.4. Then \(\textrm{Aut}(Q,C_4)\) is finite, and since \(\textrm{Aut}(Q,C_4)\) is isomorphic to a subgroup of \(\textrm{PGL}_2(\mathbb {C})\), it must be isomorphic to one of the following groups:

$$\begin{aligned}0,\mathbb {Z}_n,\mathbb {Z}_2\times \mathbb {Z}_2,D_{2n},\mathfrak {A}_4,\mathfrak {S}_4,\mathfrak {A}_5,\end{aligned}$$

where \(\mathfrak {S}_n\) (resp. \(\mathfrak {A}_n\)) is the symmetric (resp. alternating) group on n letters. Then by Lemma 2.2 the only possibilities are that \(\textrm{Aut}(Q,C_4)\) is isomorphic to \(\mathbb {Z}_2\times \mathbb {Z}_2, \mathbb {Z}_2\) or 0.

Suppose Q is in case (1). Then Q admits an action of \(\mathbb {Z}_2\times \mathbb {Z}_2\), generated by \(g_1,g_2\in \textrm{PGL}_2(\mathbb {C})\), which are given by:

$$\begin{aligned}g_1=\begin{pmatrix}1 &{} 0 \\ 0 &{} -1\end{pmatrix},g_2=\begin{pmatrix}0 &{} 1 \\ 1 &{} 0 \end{pmatrix}.\end{aligned}$$

Hence, \(\textrm{Aut}(Q,C_4)\cong \mathbb {Z}_2\times \mathbb {Z}_2\).

Suppose that Q is in case (2). Then Q admits an action of the group \(\mathbb {Z}_2\), generated by the element \(g_1\). Suppose that \(\textrm{Aut}(Q,C_4)\cong \mathbb {Z}_2\times \mathbb {Z}_2\), and let \(g\in \textrm{Aut}(Q,C_4)\) be a non-trivial element distinct from \(g_1\). Considering the standard action of \(\textrm{PGL}_2(\mathbb {C})\) on \(\mathbb {P}^1\), observe that \(g_1\) fixes the points [0 : 1] and [1 : 0], and since \(gg_1=g_1g\), we see that g must swap these points. Since g has order 2, it must be equal to either \(g_2\) or \(g_1g_2\). The threefold Q is not invariant under either of these.

Finally suppose that Q is in case (3), and suppose that \(\textrm{Aut}(Q,C_4)\) is non-trivial. Then it contains an element, g, of order 2. Since g fixes two distinct points of \(\mathbb {P}^1\), it must be equal to \(g_2\), \(g_1g_2\), or be given by a matrix of the form

$$\begin{aligned} \begin{pmatrix} 1 &{} a \\ b &{} -1 \end{pmatrix}, \text { for some } a,b\in \mathbb {C}\text { such that } \quad ab\ne -1. \end{aligned}$$

One checks that \(g_2\) nor \(g_1g_2\) leave Q invariant, and if g is given by a matrix of the above form then g(Q) is given by the equation:

$$\begin{aligned}{} & {} 4bf_0+2(1-3bc)f_1-3c(1-bc)f_2+2c^2(3-bc)f_3-4c^3f_4\\{} & {} \quad +(bc^2-2b^2c^2-4bc-c-2)f_5=0\end{aligned}$$

Clearly \(g(Q)\ne Q\), so that \(\textrm{Aut}(Q,C_4)\) has to be trivial. \(\square \)

3 Existence of the additional involution

The second half of proving Theorem 1.1 is the assertion that \(\textrm{Aut}(X)\cong \textrm{Aut}(Q,C_4)\times \mathbb {Z}_2\), which we will do in this section. The result is:

Theorem 3.1

Let X be a smooth Fano threefold in family No2.21. Then there exists an involution \(g\in \textrm{Aut}(X)\) such that \(\textrm{Aut}(X)\cong \textrm{Aut}(Q,C_4)\times \langle g\rangle \).

Proof

We proceed case-by-case, according to the classification in Theorem 2.3.

3.1 Case (1): Q is given by \(\mu (f_0+f_4)+\lambda f_2+f_5=0\)

Observe that the linear system of quadrics which contain \(C_4\) is 5-dimensional, so it is more natural to express members of family No2.21 in terms of fourfolds. Let us show how to do this. Fix the Veronese surface \(S_4\subset \mathbb {P}^5\) given by the embedding:

$$\begin{aligned} \upsilon :\mathbb {P}^2&\rightarrow \mathbb {P}^5 \\ {[}x:y:z]&\mapsto [x^2:xy:y^2:yz:z^2:xz]. \end{aligned}$$

The space of global sections of \(\mathcal {I}_{S_4}(2)\) is generated by the quadratic forms:

$$\begin{aligned} g_0&= x_3^2 - x_2 x_4,\\ g_1&= x_3 x_5 - x_1 x_4,\\ g_2&= x_5^2 - x_0 x_4, \\ g_3&= x_1 x_5-x_0 x_3, \\ g_4&= x_1^2 - x_0 x_2, \\ g_5&= x_1 x_3 - x_2 x_5, \end{aligned}$$

where \(x_0,x_1,x_2,x_3,x_4,x_5\) are homogeneous coordinates on \(\mathbb {P}^5\).

Consider the following rational map:

$$\begin{aligned} \phi :\mathbb {P}^5&\dashrightarrow \mathbb {P}^5 \\ {[}x_0:x_1:x_2:x_3:x_4:x_5]&\mapsto [g_0:g_1:g_2:g_3:g_4:g_5]. \end{aligned}$$

I claim that \(\phi \) is a birational involution. The following observation is due to I. Dolgachev: we can identify \(\mathbb {P}^5\) with the space of symmetric \(3\times 3\) matrices, upto scaling. Then under this identification, the rational map above is:

$$\begin{aligned} \phi :\mathbb {P}^5&\dashrightarrow \mathbb {P}^5 \\ \begin{pmatrix} x_0 &{} x_1 &{} x_5 \\ x_1 &{} x_2 &{} x_3 \\ x_5 &{} x_3 &{} x_4\end{pmatrix}&\mapsto \begin{pmatrix} x_3^2 - x_2 x_4 &{} x_3 x_5 - x_1 x_4 &{} x_1 x_3 - x_2 x_5 \\ x_3 x_5 - x_1 x_4 &{} x_5^2 - x_0 x_4 &{} x_1 x_5-x_0 x_3 \\ x_1 x_3 - x_2 x_5 &{} x_1 x_5-x_0 x_3 &{} x_1^2 - x_0 x_2\end{pmatrix} \end{aligned}$$

But this is the same map as taking a matrix M to its adjoint \(\textrm{adj}(M)\). Thus it follows from the relation \(\textrm{adj}(\textrm{adj}(A))=\textrm{det}(A)^{n-2}A\) for any \(n\times n\) matrix A that \(\phi \) is a birational involution.

Let \(\sigma :\widetilde{\mathbb {P}}^5\rightarrow \mathbb {P}^5\) be the blow-up of \(\mathbb {P}^5\) in \(S_4\), and let E be the exceptional divisor. Observe that for general divisors \(\widetilde{H}\in |\sigma ^*\mathcal {O}_{\mathbb {P}^5}(1)|\) and \(\widetilde{Q}\in |\sigma ^*\mathcal {O}_{\mathbb {P}^5}(2)-E|\), we have that \(\widetilde{H}\cap \widetilde{Q}\) is a smooth element of family 2.21.

Since \(\phi \) has base locus equal to \(S_4\), it lifts to a biregular involution \(g\in \textrm{Aut}(\widetilde{\mathbb {P}}^5)\) which swaps the linear systems \(|\widetilde{H}|\) and \(|\widetilde{Q}|\). Thus, the intersection \(\widetilde{H}\cap g(\widetilde{H})\) is \(\langle g\rangle \)-invariant, for any \(\widetilde{H}\). We will now show that every smooth element X of family 2.21 which is in case (1) of Theorem 2.3 is isomorphic to a subvariety of \(\widetilde{\mathbb {P}}^5\) of the form \(\widetilde{H}\cap g(\widetilde{H})\), for some hyperplane \(H\subset \mathbb {P}^5\), and therefore possesses an involution not coming from \(\textrm{Aut}(Q,C_4)\).

So fix such a threefold X. Then the quadric Q is given by the equation

$$\begin{aligned}\mu (f_0+f_4)+\lambda f_2+f_5=0,\end{aligned}$$

for some \(\lambda \in \mathbb {C}\setminus \{1,-3\}\) and \(\mu \in \mathbb {C}\setminus \{2,-2\}\) such that \(\mu ^2\ne -\lambda ^2-2\lambda +3\). Let us choose roots ab of the equations

$$\begin{aligned} (\mu +2)x^4+2\lambda -2&=0, \\ (\mu + 2)x^4+\mu -2&=0, \end{aligned}$$

respectively, so that the equation of Q becomes:

$$\begin{aligned} \frac{2 - 2b^4}{1 + b^4}(f_0+f_4)+ \frac{1-2a^4 + b^4}{1 + b^4}f_2 + f_5=0. \end{aligned}$$
(3.2)

Now consider the following hypersurfaces in \(\mathbb {P}^5\):

$$\begin{aligned} H&=\{x_0=a^2x_2+b^2x_4\},\\ Q_2&=\{g_0=a^2g_2+b^2g_4\}. \end{aligned}$$

We have that the intersections \(H\cap S_4\) and \(Q_2\cap S_4\) are smooth, so that the intersection of their strict transforms, \(\widetilde{H}\cap \widetilde{Q}_2\subset \widetilde{\mathbb {P}}^5\), is a smooth member of family 2.21. Moreover, \(Q_2=\phi (H)\), so that \(\widetilde{H}\cap \widetilde{Q}_2\) is \(\langle g\rangle \)-invariant. Consider the projective transformation \(\psi :\mathbb {P}^5\rightarrow \mathbb {P}^5\) given by the matrixFootnote 1

$$\begin{aligned} \begin{pmatrix} 1 &{} 0 &{} 0 &{} 0 &{} b^2 &{} -2b \\ 0 &{} -a &{} 0 &{} ab &{} 0 &{} 0 \\ 0 &{} 0 &{} a^2 &{} 0 &{} 0 &{} 0 \\ 0 &{} -a &{} 0 &{} -ab &{} 0 &{} 0 \\ 1 &{} 0 &{} 0 &{} 0 &{} b^2 &{} 2b \\ 1 &{} 0 &{} 0 &{} 0 &{} -b^2 &{} 0 \end{pmatrix}. \end{aligned}$$

Then the intersection of the fourfolds

$$\begin{aligned} \psi (H)&=\{x_2-x_5=0\}\\ \psi (Q_2)&=\{(1-b^4)(g_0+g_4)-a^4g_2+2(1+b^4)g_5=0\} \end{aligned}$$

is given as a subvariety of \(\mathbb {P}^4\) by Eq. 3.2, which defines X. Thus \(X\cong \widetilde{H}\cap \widetilde{Q}_2\).

It remains to show that the birational involution g commutes with the action of \(\text {Aut}(Q,C_4)\).

Consider the subgroup \(G\subset \textrm{PGL}_6(\mathbb {C})\) generated by the commuting involutions

$$\begin{aligned} \alpha :[x_0:x_1:x_2:x_3:x_4:x_5]&\mapsto [x_0:x_1:x_2:-x_3:x_4:-x_5]\\ \beta :[x_0:x_1:x_2:x_3:x_4:x_5]&\mapsto [x_0:-x_1:x_2:-x_3:x_4:x_5]. \end{aligned}$$

Then \(\alpha \) and \(\beta \) commute with the birational involution described previously:

$$\begin{aligned} \phi :[x_0:x_1:x_2:x_3:x_4:x_5]\mapsto [x_3^2 - x_2 x_4&: x_3 x_5 - x_1 x_4:x_5^2 - x_0 x_4: \\&:x_1 x_5-x_0 x_3: x_1^2 - x_0 x_2:x_1 x_3 - x_2 x_5], \end{aligned}$$

The hypersurfaces H and \(Q_2\) are G-invariant. Moreover \(S_4\) is G-invariant, so that G is isomorphic to a subgroup of \(\textrm{Aut}(Q,C_4)\). Thus since \(\textrm{Aut}(Q,C_4)\cong \mathbb {Z}_2\times \mathbb {Z}_2\) by Theorem 2.1, we have that \(\textrm{Aut}(Q,C_4)\cong G\). So we see that \(\textrm{Aut}(Q,C_4)\) commutes with the involution g.

For the remaining cases, we will compute bases for the linear system \(\mathfrak {d}\) of quadrics sections of \(Q\subset \mathbb {P}^4\) containing the curve \(C_4\) such that the corresponding rational map is an involution, and commutes with the action of \(\textrm{Aut}(Q,C_4)\).

3.2 Case (2): Q is given by \(f_0+\lambda f_2+f_5=0\)

Let us make the substitution \(\lambda =1-4s^2\), for some \(s\in \mathbb {C}\setminus \{-1,0,1\}\). Consider the rational map:

$$\begin{aligned} \iota :Q&\dashrightarrow \mathbb {P}^4 \\ [x_0:x_1:x_2:x_3:x_4]&\mapsto \left[ f_4+\frac{s^2}{2}f_2-\frac{1}{16}f_0: \frac{s}{4}f_1+sf_3: s^2f_2: sf_1: f_0\right] . \end{aligned}$$

Observe that it has base locus equal to \(C_4\), so indeed corresponds to the linear system \(\mathfrak {d}\). To see that the map \(\iota \) is a birational involution, consider the following rational parametrisation of Q,

$$\begin{aligned} p:\mathbb {P}^3&\dashrightarrow Q \\ [x_0:x_2:x_3:x_4]&\mapsto \left[ x_0x_3 : s^2x_0x_4 - s^2x_2^2 + x_2^2 - \frac{1}{4}x_2x_4 + \frac{1}{4}x_3^2 : x_2x_3 : x_3^2 : x_3x_4\right] . \end{aligned}$$

This is a rational inverse to the projection \(Q\dashrightarrow \mathbb {P}^3\) from the point [0 : 1 : 0 : 0 : 0]. Moreover, it is an isomorphism between the open subsets \(\mathbb {P}^3\setminus \Pi \) and \(Q\setminus V\), where \(\Pi \subset \mathbb {P}^3\) is the plane given by \(x_3=0\), and \(V\subset Q\) is the singular quadric surface given by the intersection of Q with the plane \(x_3=0\), this latter variety being the closure of the union of lines through [0 : 1 : 0 : 0 : 0]. Let Z be the curve \(p^{-1}(C_4)\), which is a quartic rational curve in \(\mathbb {P}^3\).

Then \(\iota (p(\mathbb {P}^3\setminus (\Pi \cup Z))\) lies in Q, and since \(p(\mathbb {P}^3\setminus (\Pi \cup Z))=Q\setminus (V\cup C_4)\) is dense in \(Q\setminus C_4\), it follows that \(\iota \) is a rational self-map of Q. To see that \(\iota \) is an involution on \(Q\setminus C_4\), observe that \(\iota \circ \iota \circ p \) is equal to the map

$$\begin{aligned} \mathbb {P}^3&\dashrightarrow Q \\ [x_0:x_1:x_2:x_3]&\mapsto \left[ x_0: \frac{(-4s^2 + 4)x_2^2 - x_2x_4 + 4s^2x_0x_4 + x_3^2}{4x_3}: x_2: x_3: x_4\right] , \end{aligned}$$

which is equal to the identity morphism on \(\mathbb {P}^3\setminus \Pi \). Thus \(\iota \circ \iota \) is equal to the identity morphism on \(Q\setminus (V\cup C_4)\), so that it is equal to the identity morphism on \(Q\setminus C_4.\)

Let us prove that \(\iota \) commutes with the action of \(\textrm{Aut}(Q,C_4)\). If \(s\ne \pm \frac{1}{2}\) then by Theorem 2.1, \(\textrm{Aut}(Q,C_4)=\langle g_1\rangle \), where \(g_1\) is the linear transformation

$$\begin{aligned}\mapsto [x_0:-x_1:x_2:-x_3:x_4].\end{aligned}$$

Then it is plain that \(\iota \) commutes with \(g_1\). If \(s=\pm \frac{1}{2}\), then Q is the quadric described in Example 1.4, and \(\textrm{Aut}(Q,C_4)\) contains subgroup isomorphic to \(\mathbb {G}_a\) consisting of automorphisms of the form

$$\begin{aligned}{}[x_0:x_1:x_2:x_3:x_4]\mapsto [x_0+4tx_1+6t^2x_2+4t^3x_3+t^4x_4&:x_1+3tx_2+3t^2x_3+x_4: \\&:x_2+2tx_3+t^2x_4:x_3+tx_4:x_4], \end{aligned}$$

for every \(t\in \mathbb {C}\). One can easily see that \(\iota \) commutes with each of these automorphisms.

3.3 Case (3): Q is given by \(f_1+f_5=0\)

Let \(\sigma \) be the rational map

$$\begin{aligned} \sigma :Q&\dashrightarrow \mathbb {P}^4 \\ [x_0:x_1:x_2:x_3:x_4]&\mapsto \left[ \frac{1}{64}f_0 - \frac{1}{4}f_1 + \frac{3}{2}f_2 + 16f_3 + 64f_4 :\right. \\&\left. :-\frac{1}{8}f_0 + \frac{3}{2}f_1 + 2f_2 + 32f_3 :f_0 - 8f_1 + 16f_2:-8f_0 + 32f_1:64f_0\right] . \end{aligned}$$

By replacing the map \(\iota \) with \(\sigma \), the map p by the rational parametrisation

$$\begin{aligned} \mathbb {P}^3&\dashrightarrow Q \\ [x_1:x_2:x_3:x_4]&\mapsto \left[ \frac{4x_1x_3 + x_1x_4 - 3x_2^2 - x_2x_3}{x_4}:x_1:x_2:x_3:x_4\right] , \end{aligned}$$

and the varieties \(\Pi \) and V with \(\{x_4=0\}\), it follows verbatim from the proof of case (2) that \(\sigma \) is a birational involution of Q with base locus equal to \(C_4\). By Theorem 2.1, \(\textrm{Aut}(Q,C_4)\) is trivial in this case, so there is nothing left to prove. \(\square \)